Copyright Final Review

Pataasin ang iyong marka sa homework at exams ngayon gamit ang Quizwiz!

(SOCRATIVE QUESTION) Vanessa is a talented video gamer developer. GreatGames is a video game developer that sometimes retains Vanessa's service as an independent contractor to work on projects they are developing, Recently one of the GreatGames' employees, Angie, tells Vanessa that GreatGames has tasked her with developing a game "that is set in a futuristic wasteland with players competing to survive. "Vanessa is intrigued by the idea, and she and Angie begin working on the game. They agree that the game will be called Dirt Wars, and its premise will feature players competing to survive in a post- apocalyptic setting. Vanessa composes about 75% of the code for the game, while Angie composes the rest. Throughout the creative process Angie and Vanessa discuss the game's development, each making substantive contributions to it. GreatGames is very pleased with the result, and they decide to release and promote Dirt Wars as part of the GreatGames catalogue. Dirt Wars is a huge success, and earns $10 million in profits. GreatGames tells Vanessa about this in an email, and says "Thanks for helping us out'. We want to compensate you fairly, so here is a check for $500,000; this is the same amount we paid Angie for her contribution. This represents the entirety of your compensation for you contribution to Dirt Wars." Vanessa feels she has been undercompensated for her contribution to Dirt Wars, and asks you what she was entitled to earn for the success of the game under copyright law. What should you tell her? (A) Vanessa should receive three times what GreatGames paid Angie because Vanessa wrote three times as much code as Angie. (B) Vanessa should receive half the profits generated by Dirt Wars because she is a joint author of the game along with Angie. (C) Vanessa should receive half the profits generated by Dirt Wars because she is a joint author of the game along with GreatGames. (D) Vanessa 's compensation is whatever GreatGames wants to pay her, since she created the game as a work made for hire for GreatGames, which alone owns the copyright in the game.

(C) Vanessa should receive half the profits generated by Dirt Wars because she is a joint author of the game along with GreatGames.

(27) All Fall Down is a best-selling novel written and published by Novelist in 2015. The novel tells the story of the discovery of an experimental plastic material that is used by a leading architectural firm to construct the world's tallest building in Los Angeles. In the novel, shortly after its completion, the building suddenly melts on the hottest day of the year, smothering thousands of people and shutting down the city for months. In 2017, Studio released The Collapsible Tower, a motion picture about the discovery of an experimental plastic material that is used to construct a Presidential Library in Washington, D.C. In the film, one of the team of scientists who made the discovery is a member of a spy network. The spy has also discovered a solvent that causes the plastic material to immediately liquefy. During the dedication of the Library, the spy applies the solvent to its structural supports, causing the building to melt, smothering the President and her cabinet. Does Novelist have a viable claim that the film infringed the copyright in his novel? (A) No, because only unprotected elements were used in the film. (B) No, because the defendant's work is in a different medium. (C) Yes, because the idea of a melting plastic building was taken from the novel. (D) Yes, because Studio is presumed to have had access to the plaintiff's novel. Questions » Topic 1 Questions: Requirements for Copyright Protection

*(A) is correct. Copyright law does not preclude others from using the ideas revealed by the author's work. See Nash v. CBS, Inc., 899 F.2d 1537, 1542 (7th Cir. 1990). And even if Studio borrowed its basic premise from Novelist's work, those ideas operate at too high a level of generality to support an infringement cause of action. At most, The Collapsible Tower and All Fall Down share only the general notion of a building being built out of material that collapses, creating disaster. This is not particularized enough expression to be copyrightable.

(39) Blooey is a famous circus clown known for his outrageous and distinctive face makeup. In 1990, Blooey came up with the highly creative design for his face makeup, which he applies to his face without assistance before each of his performances. Blooey typically performs for about an hour, and his makeup — which is composed of durable grease paint — lasts unchanged throughout his performances. Following each performance, Blooey removes his makeup, which takes him about five minutes. Recently, Blooey noticed that a clown performing under the name Reddo was using the exact same face design as Blooey. Blooey sent Reddo a polite email asking Reddo to stop using Blooey's face design, and Reddo responded by admitting that he copied his face design from Blooey but insisting that he would continue to use it. Blooey responded by telling Reddo that Blooey's face design was copyright protected, and Reddo replied that Blooey could not have an enforceable copyright in his face design. Who is right, Blooey or Reddo? (A) Blooey, because his makeup comprises an original creative work of authorship fixed in a tangible medium of expression. (B) Blooey, because courts have consistently held performers' makeup designs to be copyrightable works of authorship. (C) Reddo, because Blooey's work was unfixed. (D) Reddo, because face makeup does not fall into one of the Copyright Act's enumerated categories of works of authorship. Questions » Topic 2 Questions: Copyrightable Subject Matter

*(A) is the correct answer.* Blooey's face makeup design was clearly creative and original to him, hence its description as "outrageous and distinctive." It was also fixed in a tangible medium of expression. The human face is certainly tangible. And while his makeup was eventually removed, fixation does not require that the work persist forever. On the contrary, while ephemeral fixations may not be protected, see Cartoon Network v. CSC Holdings, Inc., 536 F.3d 121 (2d Cir. 2008) (holding that data that persists in digital medium for no longer than 1.2 seconds fails the duration element of the fixation requirement), the standard for the sufficiency of a fixation is that it persist for long enough to be perceived. Midway. Blooey's makeup easily meets this standard, since it typically is fixed for at least an hour and then ceases to exist only when Blooey takes it off. By the same token, a poet who writes a poem on paper, then decides an hour later that the poem is terrible and throws the paper into the fire has created a fixed work of authorship even though the original fixation itself has ceased to exist. (B) is incorrect because there are no court cases explicitly holding that makeup designs, on clowns or otherwise, are valid fixations solely by virtue of their being applied to the human body. While the best reading of the Copyright Act is that they are, this is an interpretation of the statutory text, not an extrapolation from case law. (C) is incorrect because, for reasons explained above, Blooey's work was fixed in a tangible medium of expression. (D) is incorrect because Blooey's makeup likely does fall into one of the categories of works of authorship listed in the Copyright Act. It is a pictorial or graphical work, see 17 U.S.C. § 102(a)(5). And even if this were not the case, the categories of works of authorship listed in the Copyright Act are inclusive, not exclusive. See id. § 102(a) ("Works of authorship include the following categories[.]"). Answers » Topic 1 Answers: Requirements for Copyright Protection

(64) Dancer is known for her powerful live dance performances, during which she performs impromptu ecstatic free-form modern dance routines, accompanied by performances of public domain classical music. Every one of her dance performances is unique, and none is planned or scripted in any way. Last year, Dancer performed her greatest work to date at Carnegie Hall. The performance was recorded with Dancer's authorization, and copies of the video were released as a DVD titled Dancer at Carnegie Hall — Live! The DVD sold well for a few months, then sales fell off because a much cheaper DVD began to circulate. Titled Carnegie Hall Hosts Dancer, Live!, the second DVD was based on an unauthorized recording of the same event. Dancer sues the makers of the second DVD for infringing her federal copyright in the choreographic work she performed at Carnegie Hall. Will she prevail? (A) Yes. Dancer will prevail because the second DVD infringes her federal copyright in the dance she performed at Carnegie Hall. (B) Yes. Dancer will prevail because the content of the second DVD infringes the audiovisual work fixed in the original DVD authorized by Dancer. (C) No. Dancer will not prevail because her dance was not fixed in a tangible medium of expression. (D) No. Dancer will not prevail because choreographic works that do not communicate a story are not copyrightable. Questions » Topic 2 Questions: Copyrightable Subject Matter

*(A) is the correct answer.* Choreographic works are copyrightable. 17 U.S.C. § 102(a)(4). Although Dancer's performance of her choreographic works was spontaneous and unplanned, even spontaneous choreographic works can be fixed in a tangible medium of expression if they are recorded by the author or under her authority. See Martha Graham School and Dance Foundation, Inc. v. Martha Graham Center of Contemporary Dance, Inc., 380 F.3d 624, 632 (2d Cir. 2004). Dancer's choreographic works that were performed in Carnegie Hall were recorded with her authorization. As a result, they were sufficiently fixed to meet the requirement for federal copyright protection. (B) is incorrect for two reasons. First, unless the audiovisual work contained in the original DVD was a work made for hire, Dancer does not own the copyright in the audiovisual work. And second, the copyright in the audiovisual work contained in the original DVD cannot prevent the creation of an independent recording of the same performance. There is no indication that the second unauthorized DVD contained any of the creative elements that made the first audiovisual work protected by copyright. (C) is incorrect because Dancer's performance was fixed in a tangible medium of expression: the authorized recording that served as the basis for the first DVD. If Dancer had not authorized any of the recordings of her Carnegie Hall performance, federal copyright law would not have protected her choreographic works, and she would have to rely on state common-law copyright. (D) is incorrect because choreographic works are clearly copyrightable, as noted in the explanation to answer (A) above, whether they communicate a story or not. Under the 1909 Act, a choreographic work could only be protected as a dramatic work, and only if it communicated a story. Under the 1976 Act, there is no such limitation.

(30) Telecom sells various ringtones that are downloaded to the customer's cell phone at the time of purchase. One of the ringtones offered by Telecom consists of one line of recorded dialogue from the motion picture Scarface spoken by actor Al Pacino, who portrayed the character Tony Montana in that film: "Say hello to my little friend!" Telecom made an audio recording of the dialogue, converted it into a ringtone, and offered it for sale in .mp3 form on their website. Telecom did all this without the permission of Universal Pictures, the owner of the copyright in the motion picture. If Universal Pictures sues Telecom for copyright infringement, Universal Pictures will likely be: (A) Unsuccessful, because motion pictures have no public performance rights. (B) Successful, because the recorded dialogue is the most memorable line of dialogue from Scarface. (C) Successful, because the motion picture Scarface is protected by copyright. (D) Unsuccessful, because the line of dialogue is not protected by copyright. Questions » Topic 1 Questions: Requirements for Copyright Protection

*(D) is the correct answer.* An individual line of dialogue from a motion picture is not protected by copyright. It does not contain sufficient originality and in some cases is merely an idea. See Murray Hill Publications, Inc. v. ABC Communications, Inc., 264 F.3d 622, 633 (6th Cir. 2001).

(6) Currently, Novelist A lives in San Francisco where he wrote The Great West Coast Novel. One year later, Novelist B, who is based in New York City, wrote and published The Great East Coast Novel. Novelist A subsequently registered the copyright in The Great West Coast Novel, which remains in manuscript form, with the Copyright Office. The contents of both novels are creative and are identical. Neither Novelist knows the other and neither has seen the other's work. Who owns the copyright in the expression of the novels? (A) Both Novelist A and Novelist B own the copyrights in their respective literary works. (B) Novelist A, because The Great West Coast Novel was created first. (C) Novelist B, because The Great East Coast Novel was published first. (D) Novelist A, because The Great West Coast Novel was registered first. Questions » Topic 1 Questions: Requirements for Copyright Protection

*(A) is the correct answer.* Copyright acknowledges the possibility of independent creation, which is the notion that two authors could independently create the same work. See Feist Publications, Inc. v. Rural Tel. Serv. Co., Inc., 499 U.S. 340, 345-46 (1991); Grubb v. KMS Patriots, L.P., 88 F.3d 1, 3 (1st Cir. 1996); Alfred Bell & Co. v. Catalda Fine Arts, Inc., 191 F.2d 99, 103 (2d Cir. 1951). "[I]f by some magic a man who had never known it were to compose anew Keats' 'Ode on a Grecian Urn,' he would be an 'author,' and, if he copyrighted it, others might not copy that poem, though they might of course copy Keats." See also, e.g., Sheldon v. Metro-Goldwyn Pictures Corp., 81 F.2d 49, 54 (2d Cir. 1936); Bryant v. Gordon, 483 F. Supp. 2d 605, 616 (N.D. Ill. 2007) (independently produced photograph entitled to copyright protection despite its identical nature to prior work).

(20) Novelist has written a lengthy murder mystery novel in French. French Novelist has assigned the English translation rights to Translator, who has yet to translate the novel. French Novelist retained all other rights. Without authorization, Publisher, an American, has translated the French novel into English and has published the English version of the novel. Publisher has entered into a settlement with French Novelist, but has refused to settle with Translator. Publisher claims that its translation was a merely mechanical process and does not constitute an actionable adaptation of the French novel. If Translator sues Publisher for copyright infringement, in the majority of jurisdictions the lawsuit will be: (A) Successful, because there is necessarily sufficient originality in Publisher's translation of the French novel into English to constitute an unauthorized derivative work. (B) Successful, because Publisher's translation of the French novel and its publication of the English translation necessarily constitute an unauthorized reproduction. (C) Unsuccessful, because the selection of an English word or phrase automatically follows from the French word or phrase being translated. (D) Unsuccessful, because Translator has yet to create an English translation and, therefore, does not have standing to sue Publisher for copyright infringement. Questions » Topic 1 Questions: Requirements for Copyright Protection

*(A) is the correct answer.* In the majority of jurisdictions, an infringement of the adaptation right can occur only if the defendant has created a work that constitutes an unauthorized derivative work that contains sufficiently original new elements. The translation of a lengthy novel from French into English is a creative endeavor that is protected by copyright as a derivative work, separate from the copyright protecting the underlying French novel. The elements of creativity involved in the translation of the French novel into English by Publisher are not merely mechanical. Such a translation involves numerous choices of words to translate the novel, selecting proper grammar and phrasing, and rendering French slang terms and idioms into understandable English terms. Had the underlying French work been more limited, such as a list of 10 words needed to order in a restaurant, the English word choices would be fewer. Such a mechanical translation would not be protected by copyright.

(24) Book Publisher has published a cookbook that contains 50 recipes featuring the use of beets. Each recipe lists the necessary ingredients for the dish as well as directions for preparation and nutritional information, without any creative narrative. Newspaper has included in its food section recipes for three of the dishes found in the cookbook. Although there are slight differences in the listing of ingredients, the recipes published by Newspaper will produce substantially the same final products as those described in the cookbook. Does Book Publisher have a valid basis for bringing a copyright infringement action against Newspaper? (A) No, because the recipes are not protected by copyright. (B) No, because the recipes constitute useful articles. (C) Yes, because the final products are substantially the same. (D) Yes, because the recipes are substantially similar. Questions » Topic 1 Questions: Requirements for Copyright Protection

*(A) is the correct answer.* Individual recipes comprising lists of required ingredients and directions for combining them to achieve final products, without expressive elaboration, constitute a "procedure" and are not entitled to copyright protection. 17 U.S.C. § 102(b). There can be a monopoly neither in the ideas for producing foodstuffs nor in the method of preparing and combining the necessary ingredients. See Publications Int'l, Ltd. v. Meredith Corp., 88 F.3d 473, 480-81 (7th Cir. 1996).

1. Musician has recently created a musical jingle she hopes to license to a greeting card company for its television advertising campaign. Musician has memorized the lyrics and music that make up the musical composition and has often sung the jingle from memory for a large number of friends at parties. Musician has not written down the lyrics or music, nor has she recorded the musical composition. Is the jingle protected by a federal copyright? (A) No, because Musician has not fixed the jingle in a tangible medium. (B) No, because Musician publicly performed the jingle prior to its registration with the Copyright Office. (C) Yes, because copyright law protects a work upon its creation. (D) Yes, because the musical composition clearly constitutes original expression. Questions » Topic 1 Questions: Requirements for Copyright Protection

*(A) is the correct answer.* The musical composition is not protected by a federal copyright because it lacks fixation. Federal copyright protection automatically attaches to a work once, and only if, it is "'fixed' in a tangible medium of expression . . . by or under the authority of the author. . . ." 17 U.S.C. § 101; Community for Creative Non-Violence v. Reid, 490 U.S. 730, 737 (1989). Since January 1, 1978, when the current federal copyright law came into effect, state law can protect only those original works of authorship that are not fixed in any tangible medium of expression. See e.g. Cal. Civ. Code § 980(a)(1).

(59) Techie created a new computer chip in 1990, and he then died in 1995. When will the period of federally enforceable exclusive rights in the structure of the computer chip expire? (A) 2000 (B) 2010 (C) 2065 (D) Computer chips are not considered copyrightable subject matter, so no federal protection is available. Questions » Topic 2 Questions: Copyrightable Subject Matter

*(A) is the correct answer.* The structure of computer chips enjoys sui generis protection by the Semiconductor Chip Protection Act of 1984, 17 U.S.C. §§ 901-14. The duration of such protection is 10 years, 17 U.S.C. § 904, therefore the federal protection of the chip will expire in 2000, because Techie created his chip in 1990. The structure of the computer chip is protected separately from the computer program embodied in the chip. The computer program is protected for the duration of works protected by copyright. (B) is incorrect because it incorrectly assumes the protection offered by the Semiconductor Chip Protection Act of 1984 lasts 20 years, which is the period of protection for patents. (C) is incorrect because it incorrectly assumes the protection offered by the Semiconductor Chip Protection Act of 1984 lasts for the life of the author plus 70 years, which is the period of protection for copyrights. (D) is incorrect because computer chips are protectable under federal law for the reasons discussed in explaining answer (A). Answers » Topic 2 Answers: Copyrightable Subject Matter

(17) has been hired to provide piano accompaniment to Singer on her upcoming concert tour. Singer has had difficulty reaching the high notes of the opening musical composition. Arranger made the musical composition easier to sing by transposing it from the key of C into the key of A. The transposition of the composition from the key of C to the key of A is easily accomplished by applying a simple, familiar formula that renders the transposition with mathematical precision. Does the transposition of the musical composition into the key of A create a derivative work that is protected by copyright? (A) No, because transposing a musical composition from one key to another is a mechanical exercise that is not protected by copyright. (B) No, because the lyrics remain the same. (C) Yes, because the notes of the resulting musical work are different from the original musical work. (D) Yes, because only a person knowledgeable in music would have the ability to transpose a musical composition from one key to another. Questions » Topic 1 Questions: Requirements for Copyright Protection

*(A) is the correct answer.* The transposition of a musical composition from one key to another, where it is done with a formula with mathematical certainty, is accomplished with no variation or creativity. Under such circumstances there is insufficient originality for copyright protection. See Jarvis v. A & M Records, 827 F. Supp. 282, 291 (D.N.J. 1993) (easily arrived-at phrases and chord progressions are usually noncopyrightable).

(49) Professor has been chosen to give the keynote address at his university's commencement. He crafts various written drafts of the address for weeks before the event. When he finally produces a written draft he is satisfied with, Professor spends days committing it to memory. By the time commencement rolls around, Professor knows the address word for word, and he delivers the address at commencement entirely from memory, reciting every word precisely as he had written it down. Student is a big fan of Professor, and digitally records, without permission, the address as Professor gives it, then transcribes the digital audio file into a Microsoft Word file and posts the text on his personal website. Professor learns of this and sues Student for copyright infringement. Will Professor prevail? (A) Yes, Professor will prevail because non-extemporaneous speeches that are composed in a written form prior to their delivery are generally considered copyrightable. (B) Yes, Professor will prevail because his memorization of the text amounts to fixation in a tangible medium of expression. (C) No, Professor will not prevail because spoken word performances do not amount to copyrightable fixations. (D) No, Professor will not prevail because speeches are a form of conversation, which is never copyrightable. Questions » Topic 2 Questions: Copyrightable Subject Matter

*(A) is the correct answer.* While courts generally do not consider conversations to be copyrightable, they have, by contrast, held that prepared speeches are copyrightable literary works. See, e.g., Jackson v. MPI Home Video, 694 F. Supp. 483, 490 (N.D. Ill. 1988) (Jesse Jackson's speech at the 1988 Democratic Party Convention found protectable as a literary work). Because Professor's speech was composed in written form beforehand, and because his delivery followed the written text precisely, it is a copyrightable work, and Student's unauthorized reproduction and display of it would amount to infringement. (B) is incorrect because no court has ever held that memorization amounts to fixation in a tangible medium of expression. (C) is incorrect because lectures based on a written text are treated differently than extemporaneous spoken word performances, which typically are not protected by federal copyright because they are not fixed in a tangible medium of expression. (D) is incorrect because courts do not treat prepared lectures as a form of extemporaneous conversation, since the former are written down in a tangible medium while the latter is not. Independently, some courts have implied that conversation may be copyrightable under certain circumstances, so the broad statement "conversation . . . is never copyrightable" inaccurately states the law. See Estate of Hemingway v. Random House, Inc., 23 N.Y.2d 341, 244 N.E.2d 250 (1969); Falwell v. Penthouse Int'l, Ltd., 521 F. Supp. 1204 (W.D. Va. 1981). Answers » Topic 2 Answers: Copyrightable Subject Matter

(55) Consider question 54. Assume that NoveltyCo releases an oversized, pink-handled magnifying glass just like the one that Judy McGee carries throughout her adventures in Author's novels. In fact, NoveltyCo concedes that it designed the magnifying glass based on the description of McGee's magnifying glass in several of Author's novels. Author contacts NoveltyCo, which did not request permission to create and market the novelty, and demands that it pay her royalties for its use of her creation. NoveltyCo refuses, and Author sues for copyright infringement. Who will prevail? (A) Author. Her copyright in the distinctive Judy McGee character extends also to identifying features of the character's persona, such as the oversized, pink-handled magnifying glass that Judy McGee uses all the time. (B) Author. Her copyright in the distinctive Judy McGee character extends also to anything associated with that character in any context. (C) NoveltyCo. Extending copyright protection to the idea of magnifying glasses would grant Author control over uncopyrightable ideas. (D) NoveltyCo. The magnifying glass is a useful article that cannot be protected by copyright. Questions » Topic 2 Questions: Copyrightable Subject Matter

*(A) is the correct answer.*Copyright protection is extended to a component part of a character that significantly aids in identifying the character. See, e.g., New Line Cinema Corp. v. Eastern Unlimited, Inc., 17 U.S.P.Q.2d 1631, 1633 (E.D.N.Y. 1989) (glove with razor fingers significantly aided in identifying Freddy Krueger character). Because Judy McGee is distinctively recognized by the oversized, pink-handled magnifying glass that she uses in all the novels, that magnifying glass is protected as part of the copyright in the character. (B) is incorrect because it overstates the scope of the law. Copyright in literary characters extends to distinctive, identifying features of those characters, but does not extend to everything that could possibly be associated with them. (C) is incorrect. Although it is true that claiming copyright in the idea of using a magnifying glass would be untenable, that is not what Author claims here. Rather, she claims copyright protection only for a particular, very distinctive magnifying glass, and only because it is uniquely associated with a character she has created. (D) is incorrect. Although the statement is correct as a general matter, a useful article can be used to flesh out a character, e.g. Indiana Jones' bullwhip and fedora hat. The useful article can be protected as part of the copyright in the character if it significantly aids in identifying the character.

(28) Young Attorney was asked by a client to draft a copyright mortgage agreement, something Young Attorney had never encountered in her short legal career. Young Attorney went to the nearest law library to look for a form book that contained such an agreement. She found a copy of the Entertainment Law Form Book that was published by Publisher. Young Attorney photocopied the pages containing the copyright mortgage agreement, brought them back to her office, inputted the language of the form agreement into her computer verbatim and filled in her client's information. Publisher recently registered the Entertainment Law Form Book with the Copyright Office. Is Young Attorney liable for infringing the copyright in the Entertainment Law Form Book? (A) Yes, because Young Attorney reproduced protected material without the permission of the copyright owner. (B) No, because under the "use doctrine" enunciated by the Supreme Court in Baker v. Selden, the use of such a work in the manner intended by the copyright owner does not constitute a copyright infringement. (C) Yes, because Young Attorney did not own the copy of the Entertainment Law Form Book that was used to make the copies of the form agreement. (D) No, because such form contracts are not protected by copyright. Questions » Topic 1 Questions: Requirements for Copyright Protection

*(B) is the correct answer. When a person uses a work protected by copyright in a manner commensurate with the manner the copyright owner intended the work to be used, an implied license will be recognized as a complete defense to a claim of copyright infringement. In this case, the intended purpose for a collection of legal forms is to provide templates to assist in the drafting of legal agreements, including the reproduction of the language published in the collection. If a person were to reproduce the entire collection of forms published as the Entertainment Law Form Book and market them under a different title, such use would constitute infringement because it was not the intent of the copyright owner to permit the copying of the forms en masse.

(15) Pursuant to an exclusive license, Television Station broadcasts a day baseball game live from the local ballpark. The live broadcast is comprised of images from several television cameras located in the ballpark from which the director of the television program selects the images to be included in the broadcast signal. It is not simultaneously recorded. Bar Owner makes a digital video recording of the game and plays it at night for his customers. Has Bar Owner infringed the federal copyright in Television Station's audiovisual work? (A) Yes, because the director's camera location and image selection are sufficiently original. (B) No, because the Television Station did not record the broadcast at the same time it transmitted the game to its viewers. (C) Yes, but only if the broadcast contains a sufficient amount of commentary by an announcer. (D) Yes, but only if the bar is larger than 3,700 square feet. Questions » Topic 1 Questions: Requirements for Copyright Protection

*(B) is the correct answer.* A live performance is fixed if it is recorded at the same time it is being transmitted. 17 U.S.C. § 101 (definition of "fixed"). See National Football League v. McBee & Bruno's, Inc., 792 F.2d 726 (8th Cir. 1986) (live broadcast of football game was fixed because it was videotaped at the same time it was did not simultaneously record its broadcast, it is unfixed and thus there was no federal copyright in it for Bar Owner to violate.

(54) Author pens a best-selling series of mystery novels that feature a central character named Judy McGee. Judy McGee is a lovable amateur sleuth who always manages to stumble onto a big case — and solve it. Her visual appearance is distinctive: she wears blue checked gingham dresses, carries an absurdly oversized, pink-handled magnifying glass to look for clues, and whenever praised, blushes modestly and utters the phrase, "Aw, jeez Louise." Author's novels become famous and the Judy McGee character becomes an admired and well-recognized cult figure. One day, Studio, a film production studio based in Manhattan, announces that it is going to develop and release a film version of a new Judy McGee story that is based on a plot written by a Studio employee, not by Author. The movie will, though, feature a Judy McGee character that faithfully includes all of her distinctive dress, magnifying glass, speech, and characteristics as described in Author's novels. Author, who has not licensed the rights to the Judy McGee character to Studio, is incensed and files for an injunction to prevent Studio from developing the movie, arguing that the use of her character amounts to copyright infringement. Assuming that the case is brought in the Southern District of New York, will Author prevail? (A) Yes, Author will prevail, because literary characters are always copyrightable as subsets of the literary works in which they appear. (B) Yes, Author will prevail because Judy McGee is sufficiently fleshed out with distinctive physical and personality traits to merit copyright protection. (C) No, Author will not prevail, because Judy McGee is merely a plot device and does not constitute the story being told. (D) No, Author will not prevail, because literary characters, unlike characters depicted in graphic form, are not copyrightable. Questions » Topic 2 Questions: Copyrightable Subject Matter

*(B) is the correct answer.* All circuits, except the Ninth Circuit, have clearly held that literary characters depicted in novels are copyrightable as long as they are sufficiently distinctive. That is, they must be sufficiently fleshed out with recognizable physical traits, backstory, or personality traits in order to be copyrightable. Filmvideo Releasing Corp. v. Hastings, 509 F. Supp. 60 (S.D.N.Y. 1981), aff'd, 668 F.2d 91 (2d Cir. 1981); Silverman v. CBS, Inc., 870 F.2d 40, 50 (2d Cir. 1989). Here, the Judy McGee character is clearly distinctive in terms of her appearance and behavior, so she is copyrightable, and Studio's use of the character without seeking a license from Author amounts to infringement. (A) is incorrect because literary characters are not always copyrightable. Rather, they must be sufficiently well developed to be distinctive and recognizable in order to merit copyright protection. See Nichols v. Universal Pictures Corp., 45 F.2d 119, 121 (2d Cir. 1930). (C) is incorrect because it refers to the minority view, the Ninth Circuit test for protectability of literary characters. This test sets a higher bar. It looks to whether the character at issue represents the actual story being told or is merely a plot device used to move the story along. Only if the character is sufficiently central to the narrative does it merit protection under this test. See Warner Bros. Pictures, Inc. v. Columbia Broad. Sys., Inc., 216 F.2d 945, 950 (9th Cir. 1954) (literary character Sam Spade held not protected by copyright); Rice v. Fox Broad. Co., 330 F.3d 1170, 1176 (9th Cir. 2003) ("Mystery Magician" held to be merely a chess piece in the game of telling the story of how magic tricks are performed). Some courts in the Ninth Circuit have, however, moved away from exclusively applying the "Sam Spade" test. See Shaw v. Lindheim, 908 F.2d 531 (9th Cir. 1990); Metro-Goldwyn-Mayer, Inc. v. American Honda Motor Co., Inc., 900 F. Supp. 1287 (C.D. Cal. 1995). (D) is incorrect because literary characters are copyrightable if they meet the standards described in the discussion of answer (B) above.

(7) Artist has created a new work. It is a five-pointed star cut out of cardboard. Artist has cut a square out of the middle of the star, permitting the insertion of a small photograph. Is Artist's creation protected by copyright? (A) No, because the square cut-out renders the star an unprotected useful article. (B) No, because the star is a common shape. (C) Yes, because the material used to create a work does not affect its copyrightability. (D) Yes, because it is a sculptural work. Questions » Topic 1 Questions: Requirements for Copyright Protection

*(B) is the correct answer.* Copyright does not protect common geometric shapes. See Bailie v. Fisher, 258 F.2d 425 (D.C. Cir. 1958); Atari Games Corp. v. Oman, 888 F.2d 878 (D.C. Cir. 1989); 37 C.F.R. § 202.1 ("familiar symbols or designs" and "mere variations of typographical ornamentation, lettering or coloring" are not subject to copyright protection). Here, the work consists simply of a common shape with another common shape cut out of it. This mere juxtaposition of two common shapes does not rise to the level of original copyrightable expression.

(52) Cartoonist creates a series of comic books based on a character he calls "SuperCop," who is a bumbling, feckless policeman by day, but morphs into a crime fighter with super powers by night. SuperCop lives in present-day Urbania, a city modeled on Detroit, and typically fights familiar, realistic enemies such as drug kingpins and serial murderers. After the series has become very successful, Cartoonist learns of the recent publication of a comic book series titled SuperCop in Outer Space, which portrays a graphically identical SuperCop character living on a futuristic space station and fighting strange creatures from outer space. Cartoonist contacts Publisher, the company that publishes SuperCop in Outer Space, and complains about its unauthorized use of his character. Publisher responds, "Sure, it's the same character, but it's not infringement because we didn't copy any of your stories, and in our series SuperCop is in a completely different setting." Unconvinced, Cartoonist sues Publisher for copyright infringement. Will he prevail? (A) Yes, Cartoonist will prevail because his original creation of the SuperCop character gives him the right to control all subsequent literary or pictorial representations of police performing extraordinary feats. (B) Yes, Cartoonist will prevail because Publisher's use of SuperCop is a verbatim copy of the attributes of Cartoonist's character that comprises an original work of authorship. (C) No, Cartoonist will not prevail because Publisher's use of SuperCop sufficiently decontextualized the character from its original setting to such a degree that the taking does not appropriate any protected expression. (D) No, Cartoonist will not prevail because graphic characters are not copyrightable. Questions » Topic 2 Questions: Copyrightable Subject Matter

*(B) is the correct answer.* Courts have consistently held that characters are copyrightable as long as they are represented with sufficient distinctness, and are not represented in a generic or commonplace manner. See Nichols v. Universal Pictures Corp., 45 F.2d 119, 121 (2d Cir. 1930) ("[T]he less developed the characters the less they can be copyrighted, that is the penalty an author must bear for marking them too indistinctly."). Characters depicted graphically (i.e., visually rather than in literature) are particularly likely to merit copyright protection. See Walt Disney Prods. v. Air Pirates, 581 F.2d 751 (9th Cir. 1978). Here, the SuperCop character has a clearly recognizable image that has been appropriated without permission, so Cartoonist's infringement suit against Publisher will likely prevail. (A) is incorrect because it describes the scope of Cartoonist's rights in SuperCop too broadly. Cartoonist enjoys statutorily enumerated exclusive rights in the character, but does not have a broad right to control every subsequent graphic depiction of a heroic policeman. (C) is incorrect because placing a copyrighted literary character in a different context does not obviate a plaintiff's claim for infringement. The copyright in the character persists as long as the character is reproduced without permission, regardless of the context in which it is featured. See Walt Disney Prods. v. Air Pirates, 581 F.2d 751 (9th Cir. 1978); Walt Disney Prods. v. Filmation Assocs., 628 F. Supp. 871 (C.D. Cal. 1986). (D) is incorrect because courts have consistently held that graphical characters are copyrightable. See Metro-Goldwyn-Mayer v. Am. Honda Motor, 900 F. Supp. 1287 (C.D. Cal. 1995).

(35) has published a romantic novel concerning the detailed planning and operation of a dinosaur zoo. The novel begins with the selection of a remote island on which to build the zoo, the creation of a dinosaur nursery, the use of uniformed guards and tall electric fences for security, and the design of automated tours for those attending the zoo. Subsequent to the publication of Novelist's work, Publisher releases a children's book about an adventure park that contains dinosaurs. The adventure park in the children's book is set on a prehistoric island far from the mainland where the dinosaurs are hatched in a nursery. At the park, children ride on automated vehicles alongside tall electric fences that corral the dinosaurs. Throughout the story, uniformed guards accompany the children. Novelist claims that Publisher's children's story infringes the copyright in her romantic novel. Will Novelist's infringement action be successful? (A) Yes, because strikingly similar elements appear in both works. (B) No, because the elements that have been identified by Novelist constitute scènes à faire. (C) Yes, because the children's story is infringing even if the details of the elements in question differ between the two works. (D) No, because the two works are directed at different audiences. Questions » Topic 1 Questions: Requirements for Copyright Protection

*(B) is the correct answer.* Elements or sequences of events that necessarily result from a chosen concept do not enjoy copyright protection. The setting of a dinosaur zoo or adventure park, with electrified fences, automated tours, dinosaur nurseries, uniformed workers, and placing dinosaurs on a prehistoric island far from the mainland "are classic scènes à faire that flow from the uncopyrightable concept of a dinosaur zoo." Williams v. Crichton, 84 F.3d 581, 589 (2d Cir. 1996).

(65) Photog takes a photograph of a wet kitten in a sink and prints it on shirts that he sells via the Internet. The sales of the shirts are a great success. Soon, Knockoff starts a website from where he sells an identical shirt, including its unique button holes and an exact copy of Photog's kitten photograph, for a fraction of the price charged by Photog. Photog sues Knockoff for copyright infringement. Will he succeed? (A) Yes. Photog will succeed because by appending an original photograph to a functional object like a shirt, he acquires exclusive rights over the entire shirt as well as what is depicted on it. (B) Yes. Photog will succeed, but only with respect to Knockoff's reproduction and distribution of the photograph contained on the shirt. (C) No. Photog will not succeed because clothing is not copyrightable. (D) No. Photog will not succeed because by appending a copyrightable image to a non-copyrightable useful object, he forfeits copyright protection in the former.

*(B) is the correct answer.* Photog has an enforceable copyright in his original photograph. However, the infringement claim applies to the photograph only; Photog does not have any copyright interest in the shirt, which is an uncopyrightable useful object. See Poe v. Missing Persons, 745 F.2d 1238 (9th Cir. 1984) (clothing not protectable by copyright). But Knockoff's reproduction of the image on a shirt, as opposed to selling copies of it as a photograph, does not negate Photog's argument that his exclusive right of reproduction was infringed. Although courts consider clothing an uncopyrightable useful object, aesthetic features of utilitarian objects that are physically or conceptually separable from the objects on which they appear, such as, here, the kitten photograph on the shirts, are considered copyrightable. See Mazer v. Stein, 347 U.S. 201 (1954); Esquire, Inc. v. Ringer, 591 F.2d 796, 804 (D.C. Cir. 1978). (A) is incorrect because Photog's copyright is limited to his original photograph and does not extend to the utilitarian object on which it is depicted. (C) is incorrect. Although clothing is indeed not copyrightable, where, as here, a separate image or design is depicted on clothing, that image is copyrightable. (D) is incorrect because the copyrightable status of protectable images that have been appended to utilitarian objects is preserved by the separability doctrine. Answers » Topic 2 Answers: Copyrightable Subject Matter

(2) Musician has recently composed an original musical composition that she recorded by singing the composition herself, and recording her singing it on her personal digital audio recorder. She has not performed the composition for anyone, nor has she shared the recording with anyone. She has not placed a copyright notice on the digital recording. Is the musical composition protected by a federal copyright? (A) Yes, because the musical composition is sufficiently fixed in a copy. (B) Yes, because the musical composition is sufficiently fixed in a phonorecord. (C) No, because the recording is not accompanied by a proper copyright notice. (D) No, because no one can see the musical composition in the digital recording. Questions » Topic 1 Questions: Requirements for Copyright Protection

*(B) is the correct answer.* The musical composition is fixed in the tangible medium of the digital recording Musician has made. The digital recording is a phonorecord because it is an object containing sounds other than those accompanying an audiovisual work that can be perceived with the aid of a machine or device. See 17 U.S.C. § 101 (defining "phonorecord" to refer to fixations of sound recordings). The prerequisites of federal copyright have been met and federal copyright protection attaches immediately.

(58) Programmer writes and registers the source code for MacroSoft Bird, a program that allows birdwatchers to catalog their avian sightings. The program is a big success, and soon after its release, a rogue programmer named Pirate releases a similar program. Pirate's program, titled Birds in Flight, allows birdwatchers to catalog their avian sightings, but also provides information that enables birdwatchers to identify different species as well as connect with birdwatchers around the globe. Programmer learns that Pirate created Birds in Flight by cutting and pasting the entirety of the source code for MacroSoft Bird into a file, and then appending additional portions of original source code that enable the other features of the program. Programmer wants to sue Pirate for infringement of the MacroSoft Bird source code as well as the program's bird-cataloging feature. What will the result be? (A) Programmer will prevail with respect to both the source code and the cataloging feature. (B) Programmer will prevail with respect to the source code only. (C) Programmer will prevail with respect to the cataloging feature only. (D) Programmer will fail with respect to both aspects of his infringement suit. Questions » Topic 2 Questions: Copyrightable Subject Matter

*(B) is the correct answer.* The source code that comprises computer programs is copyrightable, Apple Computer, Inc. v. Franklin Computer Corp., 714 F.2d 1240, 1248-49 (3d Cir. 1983), but the functionality of computer programs is not, because it amounts to an unprotectable idea. See Computer Assocs. Int'l, Inc. v. Altai, Inc., 982 F.2d 693 (2d Cir. 1992). Programmer can successfully sue Pirate for infringing the source code, which Pirate copied verbatim, but the idea of a program that catalogs bird sightings operates at too high a level of generality to be protectable. The cataloging function may be protectable by patent law, if it is sufficiently novel, nonobvious and useful, but not by copyright law. (A), (C), and (D) are incorrect because they falsely assume either that source code is not protectable or that ideas are. Answers » Topic 2 Answers: Copyrightable Subject Matter*

(63) Polly Math is a multitalented artist who writes the melody and lyrics to a tune titled Only Me and then records an a cappella version of the tune using her home recording studio. What copyrights does she own, and in what categories of works of authorship are they included? (A) Polly owns copyrights in the melody and lyrics of Only Me, which comprise a single musical work. (B) Polly owns the copyright in the melody and lyrics of Only Me, which comprise a single musical work; and the copyright in the recording of her performance of Only Me, which comprises a sound recording. (C) Polly owns the copyrights in the lyrics of Only Me as a musical work, and the copyright in the melody and her performance of Only Me, which comprise a single sound recording. (D) Polly owns the copyrights in the melody, lyrics, and performance of Only Me, which comprise three separate musical works. Questions » Topic 2 Questions: Copyrightable Subject Matter

*(B) is the correct answer.* Though the Copyright Act does not define "musical works," see 17 U.S.C. § 102(a)(2), courts have construed this term to include the harmony, melody, and accompanying words that comprise a musical composition. See Mills Music, Inc. v. Arizona, 187 U.S.P.Q. 22 (D. Ariz. 1975), aff'd per curiam, 591 F.2d 1278 (9th Cir. 1979). By contrast, the recording of a performance of a musical work constitutes a fixation of sounds in a phonorecord, which is a sound recording. See 17 U.S.C. § 101 (definition of "sound recordings"). Thus, Polly Math owns a copyright in both elements of the musical work, melody and lyrics, which together comprise Only Me, while her recorded performance of Only Me constitutes a sound recording in which she owns a separate copyright. (A) is incorrect because it fails to recognize that Polly Math also owns the copyright in the sound recording of Only Me. (C) is incorrect because it wrongly states that Polly Math owns the copyright in the melody of Only Me as a sound recording, not a musical work. (D) is incorrect because it wrongly states that Polly Math owns the copyright in the recorded performance of Only Me as a musical work. Further, the recording of a performance of Only Me would constitute a sound recording, not a musical work. The performance of a work is not copyrightable subject matter. The work being performed and the recording of that performance are copyrightable subject matter.

(9) A motion picture was recently released for general exhibition that depicted a used guitar store on the moon after the people of Earth had colonized it. Two years later, a dramatic television series that depicted a small alien civilization engaged in intergalactic space travel included a stop on the moon where one of the aliens purchased a guitar from a used guitar store. The signage, size, and contents of the stores differed substantially, as did the store's proprietor and customers. Has the television series infringed on the copyright of the motion picture? (A) Yes, because it is the similarities between the two works that are at issue, not the dissimilarities. (B) Yes, because the idea of a used guitar store on the moon is novel. (C) No, because television is a different medium from motion pictures. (D) No, because once the motion picture was released to the public, anyone has the right to use ideas found in the motion pictur Questions » Topic 1 Questions: Requirements for Copyright Protection

*(D) is the correct answer.* Copyright law does not preclude others from using the ideas revealed by the author's work. See Nash v. CBS, Inc., 899 F.2d 1537, 1542 (7th Cir. 1990).

(23) Composer is the author of a musical play titled Flash, Trash & Cash. The musical play is based on the true story of an aspiring singer who tragically dies in an avalanche. Composer is concerned that Flash, Trash & Cash is also the title of an earlier published fictional novel, protected by copyright, that is based in the milieu of the exotic dance clubs that populate the east coast of the United States. Should Composer be concerned about a possible claim of copyright infringement resulting from her choice of title for her musical play? (A) No, the subject matter of the two works is not substantially similar. (B) No, titles of books are not protected by copyright. (C) Yes, the titles are identical. (D) Yes, the book's title was used prior to the title's use in the musical play. Questions » Topic 1 Questions: Requirements for Copyright Protection

*(B) is the correct answer.* Titles of books are not protected by copyright, because they are not sufficiently original for copyright protection or are deemed to constitute mere ideas. Although a well-known book title may be protected as a trademark, it cannot be protected by copyright. See, e.g., Shaw v. Lindheim, 919 F.2d 1353, 1362 (9th Cir. 1990) (title of "The Equalizer" television series not protected by copyright); Trenton v. Infinity Broadcasting Corp., 865 F. Supp. 1416, 1426 (C.D. Cal. 1994) (title of "Loveline" radio program not protected by copyright); Morgan Creek Prods Inc. v. Capital Cities/ABC Inc., 22 U.S.P.Q.2d 1881, 1883 (C.D. Cal. 1991) (title of film "Young Guns" cannot be protected by copyright); Perma Greetings, Inc. v. Russ Berrie & Co., Inc., 598 F. Supp. 445, 448 (E.D. Mo. 1984) ("Cliched language, phrases and expressions conveying an idea that is typically expressed in a limited number of stereotypic fashions, are not subject to copyright protection."). But see Universal City Studios, Inc. v. Kamar Industries, Inc., 217 U.S.P.Q. 1162, 1166 (S.D. Tex. 1982) ("I Love You, E.T." and "E.T. Phone Home" protected by copyright).

(18) In 1980, Verso conceives and writes the haiku Leaves of Autumn on rice paper in ink. Verso deems the work inadequate and puts it in his desk drawer. One day, Verso's friend Poetaster snoops around in Verso's office and runs across Leaves of Autumn. Poetaster copies the work verbatim onto a piece of note paper, and later publishes it under his own name in Poetry magazine. Verso is incensed that Poetaster has copied his haiku, and sues Poetaster for copyright infringement. Poetaster responds by showing that in 1979, a poet named Quarto wrote a haiku titled Rainbow Leaves that is identical to Verso's haiku. Verso admits that the 1979 and 1980 haikus are identical, but truthfully avers that when he wrote Leaves of Autumn in 1980, he had never seen or heard of Quarto's Rainbow Leaves. Who will prevail in the infringement suit, Verso or Poetaster? (A) Verso will prevail because Leaves of Autumn possesses the necessary degree of creativity sufficient to meet the standard of originality. (B) Verso will prevail because Leaves of Autumn possessed the necessary degree of creativity sufficient to meet the standard of originality, and that creativity originated with Verso. (C) Poetaster will prevail because Verso's haiku was identical to a previously created haiku. (D) Poetaster will prevail because Verso will not be able to prove that Poetaster copied Verso's Leaves of Autumn, as opposed to Quarto's Rainbow Leaves Questions » Topic 1 Questions: Requirements for Copyright Protection

*(B) is the correct answer.* Verso will prevail not only because Leaves of Autumn is creative on its own terms, but also because that creativity originated with Verso. The originality requirement is satisfied if two tests are met: independent creation and a modicum of creativity. Proving that the work is the result of independent creation and, therefore, original, is required to meet the burden of proving copyright validity. See Reader's Digest Ass'n v. Conservative Digest, Inc., 821 F.2d 800, 806 (D.C. Cir. 1987) ("Originality" means "only that the work owes its origin to the author — i.e., that the work is independently created rather than copied from other works."). However, the concept of independent creation also recognizes the possibility that the defendant did not copy the plaintiff's work, but created the defendant's work independently. If that can be proven, the defendant will not be held liable for copyright infringement.

(36) B-Loko is a popular energy drink made and owned by B-Loko, Inc., that contains very high doses of both caffeine and sugar. Every can of B-Loko contains the following warning: "This beverage contains an amount of caffeine that significantly exceeds the Food and Drug Administration's Recommended Daily Allowance (RDA) of caffeine. You may experience some adverse symptoms upon drinking this beverage, such as sweating, redness in the face and neck, and an elevated heartbeat. Limiting your caffeine intake will help to ameliorate these symptoms." Recently, a competitor of B-Loko, produced and owned by a company called B-Krazy, Inc., entered the market. This beverage, called B-Krazy, also contains very high levels of both caffeine and sugar, though it also adds a supplement called tuanine that has been said to increase energy. Cans of B-Krazy feature a warning label that uses the exact same language as the B-Loko warning. B-Loko, Inc. sues B-Krazy, Inc., arguing that their B-Krazy, Inc.'s unauthorized use of the warning from cans of B-Loko infringes B-Loko, Inc.'s copyright in that material. B-Krazy, Inc. admits that they copied the warning verbatim, but argues that they cannot be held liable for copyright infringement because the warning is not copyrightable. Who will prevail on this argument? (A) B-Loko, because B-Krazy's use of a very similar name and its admission that it used the warning without authorization show that it had clear intent to infringe. (B) B-Loko, because the warning met the very low standard for copyright originality. (C) B-Krazy, because while the warning did possess some spark of creativity, it was insufficient to meet the high standard for copyright originality. (D) B-Krazy, because the warning lacked any creativity or originality, and thus did not qualify for copyright protection. Questions » Topic 1 Questions: Requirements for Copyright Protection

*(B) is the correct answer.* While copyright law does require some "spark of creativity" in order to meet the originality requirement for copyrightability, this has traditionally been held a very low standard that is easy to meet. See Feist Publications v. Rural Tel. Serv. Co., 499 U.S. 340, 345 (1991) ("the requisite level of creativity is extremely low; even a slight amount will suffice"). The B-Loko warning label is not highly creative, but does seem to possess enough distinctive phrasing in terms of its grammatical structure and word choice that it meets this low bar — but only just.

(46) was recently fired from Company, and is none too happy about it. Late one night, he sneaks into Company's headquarters with revenge on his mind. He rifles through Company's files and finds several documents: a brochure containing information about Company's secret new product line that it is planning to release in two months; internal memos discussing Company's strategy to bilk employees out of health care; and a video from a Christmas party, produced by a professional videographer as a work made for hire, that shows Company executives sloppily drunk. Employee posts all this material on his website, and a scandal ensues. Company sues Employee for, among other things, copyright infringement, and seeks an injunction ordering Employee to take the offending material offline. Will Company succeed, and if so, with respect to which documents? (A) Company will succeed, but only with respect to the brochure and the video, because the memo is not copyrightable subject matter. (B) Company will succeed, but only with respect to the brochure and the memo, because the video is not copyrightable subject matter. (C) Company will succeed with respect to the brochure, the memo, and the video, because all of them are copyrightable subject matter. (D) Company will not succeed with respect to any of the documents because none of them amount to copyrightable subject matter. Questions » Topic 2 Questions: Copyrightable Subject Matter

*(C) is the correct answer.* All of the expression contained in the material taken by Employee amounts to copyrightable subject matter. The brochure, video, and memorandum all contain at least a modicum of creative expression, and are fixed in tangible media of expression. While these materials may not be typical subjects of copyright, the fact that they were produced in a business context does not deprive them of protection. See, e.g., Khandji v. Keystone Resorts Mgmt., Inc., 140 F.R.D. 697 (D. Colo. 1992) (settlement brochure is copyrightable); Rexnord, Inc. v. Modern Handling Sys., Inc., 379 F. Supp. 1190 (D. Del. 1974) (manufacturer's catalogue is copyrightable). (A), (B), and (D) are incorrect because they state that some of the material at issue is not copyrightable.

(3) Poet wrote a poem honoring the first butterfly of the season. After writing the poem by hand in a notebook, Poet placed it in a desk drawer. A month later, Poet recited the poem to members of his local literary society who unanimously commented that the poem was poorly crafted. That evening, in a fit of embarrassment, Poet ripped the page containing the poem from the notebook and burned the page in his fireplace. After Poet burned the poem, is there still a valid federal copyright protection in the poem? (A) No, because the poem was poorly crafted. (B) No, because the poem is no longer fixed in a tangible medium of expression. (C) Yes, because the poem had been fixed in a tangible medium of expression. (D) Yes, because Poet believed the poem had literary merit at the time he wrote it. Questions » Topic 1 Questions: Requirements for Copyright Protection

*(C) is the correct answer.* At the moment the poem was fixed on the notebook page, it automatically received federal copyright protection. The destruction of the only copy of the poem creates problems in proving infringement or depositing a copy of the work with the Copyright Office, but these difficulties do not negate the federal copyright protection accorded the work from the date of its creation in a tangible form. See Peter Pan Fabrics, Inc. v. Rosstex Fabrics, Inc., 733 F. Supp. 174, 177 (S.D.N.Y. 1990) (once work is fixed, its subsequent destruction does not vitiate its copyright).

(57) Animator created the Mice-a-Roni comic strip in the mid-twentieth century. Mice-a-Roni featured Mikey the Mouse, a lovable anthropomorphic mouse, as its main character. Assume for the purposes of this question that the Mikey character is clearly copyrightable. Animator's Mice-a-Roni comic strip enjoyed some success and ran in newspapers for about 20 years, when it fell out of popularity and Animator ceased to produce it. Animator did not include a copyright notice on his comic strip during the first few years that he produced Mice-a-Roni, but as the series began to enjoy some popularity, he started to include the necessary formalities. As a result, the earliest five years of the Mice-a-Roni comics have fallen into the public domain. The subsequent 15 years of the comic strip have contained a copyright notice. One day, Animator learns that Publisher has released a book titled Classic Comix of the 50s Re-imagined in which he portrays various old comic characters, including Mikey the Mouse, in a modern setting. Publisher did not seek permission to use the Mikey character. Animator sues Publisher for copyright infringement. What result? (A) Animator will prevail. As long as a single episode of Mice-a-Roni is protected by copyright, the Mikey character is protected by copyright as well. (B) Animator will prevail. The copyright protection period for graphic characters is limitless. (C) Animator will not prevail. Characters fall into the public domain when the work of authorship in which they initially appeared falls into the public domain. (D) Animator will not prevail. A court would regard Animator's failure to comply with copyright formalities during the early years of Mice-a-Roni as a form of laches, which means Animator forfeits any right to enforce his copyright in Mikey the Mouse later on. Questions » Topic 2 Questions: Copyrightable Subject Matter

*(C) is the correct answer.* Characters, including graphic characters, fall into the public domain as soon as any work in which they are depicted falls into the public domain. A character will enter the public domain when the work in which the character was initially sufficiently delineated enters the public domain. See Klinger v. Conan Doyle Estate, Ltd., 755 F.3d 496 (7th Cir. 2014) (holding that Sherlock Holmes character is in the public domain because initial versions of the character are in the public domain); Silverman v. CBS, Inc., 870 F.2d 40, 50 (2d Cir. 1989). Mikey the Mouse thus became public domain material when the first Mice-a-Roni comic became part of the public domain. Mikey the Mouse is no longer protected, so Publisher is free to use it without a license. That other elements of the series, such as new stories, characters, images and dialogue, remain protected by copyright does not matter. (A) is incorrect. It gets the law exactly backwards: if any one literary work in which a character is sufficiently well delineated becomes part of the public domain, then the character becomes part of the public domain as well. It does not matter if there are other works depicting the public domain character that contain other copyrighted material. (B) is incorrect because it misstates the law. The period of protection for characters depicted in any works of authorship is no longer than it is for those works of authorship. (D) is incorrect. The reason that Mikey the Mouse is not protected is that Animator failed to preserve his copyright in the character, not because a court would look askance at his failure to follow those formalities on some equitable theory. Publisher will not have to rely on an affirmative defense, such as laches, when the subject matter at issue, the character of Mikey the Mouse, is no longer protected by copyright.

(14) screen installed in a local public museum. The screen features a random combination of images sent to it by 10 computers, each with a different bank of images contained on its hard drive. Photographer has taken unauthorized photographs of the images appearing on the plasma screen and sells them in his art gallery. Artist sues Photographer for copyright infringement. Artist's lawsuit will be: (A) Unsuccessful, because the images comprising the digital artwork are not fixed in a single object or device. (B) Unsuccessful, because the work has been dedicated to the public domain. (C) Successful, because each image is capable of being reproduced. (D) Successful, because the photographs taken by Photographer have fixed the images in a tangible medium. Questions » Topic 1 Questions: Requirements for Copyright Protection

*(C) is the correct answer.* Every image that can be seen on the plasma screen can be obtained from one of the computers used in the installation. The fact that the images are reproduced from different sources does not mean the individual images or the combination of images appearing on the screen are not fixed. See Midway Mfg. Co. v. Arctic Int'l, Inc., 547 F. Supp. 999 (N.D. Ill.), aff'd, 704 F.2d 1009 (7th Cir. 1982) (audiovisual work component of video game properly fixed in several ROM chips).

(43) After submitting her latest manuscript for publication, Author decides that she wants to change the ending. She writes a letter to Publisher outlining the ending and asking that Publisher not send the book to press until she revises the manuscript. Under pressure to get the book out, Publisher releases the manuscript in its original form, but includes Author's letter verbatim as an epilogue. Author, who never gave permission for Publisher to publish anything but the manuscript, is angry and sues Publisher for infringing her exclusive rights in the letter. Will she prevail? (A) No, because letters are expository, and thus do not count as "literary works" as defined by the Copyright Act. (B) No, because by sending the letter to Publisher, Author granted an implied license to publish its contents. (C) Yes, because the publication of the letter violates her exclusive rights in a copyrightable work of authorship. (D) Yes, because the final chapter included in the letter is literature, and this makes the letter sufficient to meet the definition of "literary works" in the Copyright Act. Questions » Topic 2 Questions: Copyrightable Subject Matter

*(C) is the correct answer.* Letters may not be regarded as literature in a colloquial sense, but this is irrelevant to the question whether they are considered works of authorship for the purpose of copyright analysis. Courts have consistently held that letters are copyrightable so long as they contain sufficient original expression. See, e.g., Salinger v. Random House, Inc., 811 F.2d 90, 94-95 (2d Cir. 1987) (author of the letter owns the copyright in its contents). So Publisher's unauthorized publication of Author's copyrighted material amounts to infringement. (A) is incorrect because expository materials certainly do count as "literary works" within the meaning of the Copyright Act. The term "literary works" is not given a narrow or colloquial construction to refer only to works of literature. Rather, courts construe it broadly to refer to any work that is composed in words or numbers. See 17 U.S.C. § 101 ("Literary works") ("'Literary works' are works, other than audiovisual works, expressed in words, numbers, or other verbal or numerical symbols or indicia."). (B) is incorrect because Author explicitly asked Publisher not to publish the existing manuscript or new ending until after she had the opportunity for revision. There is no way this language could be construed as implied or explicit permission to publish the copyrighted material contained in the letter. (D) is incorrect for much the same reasons given for answer (A). Remember, a work need not amount to "literature" to count as a literary work or a work of authorship within the meaning of the Copyright Act.

(60) User is the owner of an authorized copy of DethKlok 2000, a popular new video game, and proceeds to play it on his home computer. Any time User activates the game, his computer automatically copies several of the files, written in object code (machine language), that comprise DethKlok 2000 into its Random Access Memory (RAM). Does this duplication of the object code that comprises DethKlok 2000 amount to copyright infringement? (A) Yes. The unauthorized duplication of any literary work, including a computer program written in object code, infringes the copyright in that work. (B) No. While computer source code may be copyrightable, a computer program written in object code is not, so this copying is not actionable. (C) No. This kind of copying appears to be prima facie infringement, but the Copyright Act includes an exception to permit it. (D) No. Because User did not intend for any copying to take place, there can be no infringement. Questions » Topic 2 Questions: Copyrightable Subject Matter

*(C) is the correct answer.* The 1976 Copyright Act provides for the unauthorized copying of a computer program by the owner of the computer program when it is an essential step in the utilization of the program in conjunction with the computer or when the copy is used for archival purposes. 17 U.S.C. § 117. See Allen-Myland, Inc. v. Int'l Business Machines Corp., 746 F. Supp. 520, 536 (E.D. Pa. 1990). (A) is incorrect because, although object code is a copyrightable literary work, this answer fails to take into account the section 117 exception. (B) is incorrect because object code is copyrightable. See Apple Computer, Inc. v. Franklin Computer Corp., 714 F.2d 1240, 1248-49 (3d Cir. 1983) (computer programs are copyrightable subject matter regardless of whether they are expressed in human-readable source code or machine-readable object code); Apple Computer, Inc. v. Formula Int'l, Inc., 725 F.2d 521, 523 (9th Cir. 1984). (D) is incorrect because User's intent is irrelevant; copyright infringement is a strict liability tort.

(29) Author wrote Shadow Land, a biographical literary work about the life of Frances Farmer, an independent, free-spirited actress who fought against the conventions of her time. In Shadow Land, Author described how Farmer was institutionalized by her mother, underwent a lobotomy, and emerged from the psychiatric facility with her spirit broken. Shadow Land was marketed as the "true story" of Frances Farmer. After Shadow Land was published, Producer produced The Farmer Story, a motion picture based on the life of Francis Farmer. The motion picture presented a dramatization of Farmer's lobotomy and its aftermath that closely resembled the description of those events in the literary work. Author has now sued for copyright infringement. Producer is defending the lawsuit on the basis that she only used the relevant facts that were published in the literary work. Author argues that the lobotomy and its aftermath were not facts, but fictional elements that Author had created. Will Author be successful with his copyright infringement claim? (A) No, because facts are not protectable by copyright. (B) Yes, because fictional elements are protectable by copyright. (C) No, because fictional elements that are presented by the copyright owner as fact will be treated as though they were facts. (D) Yes, because the elements at issue were important plot points in the literary work. Questions » Topic 1 Questions: Requirements for Copyright Protection

*(C) is the correct answer.* The doctrine of copyright estoppel is an exception to the general rule that fictional elements in a work are protectable under copyright law. Once a plaintiff's work has been held out to the public as factual, the copyright owner cannot then claim that the work is, in actuality, fiction and thus entitled to copyright protection. See Houts v. Universal City Studios, Inc., 603 F. Supp. 26, 28 (C.D. Cal. 1984).

(32) Two years ago, Millennium Studios released its motion picture FangTastic!. The film deals with a modern day vampire who lives in a trailer park and details her efforts to have a romantic relationship with a human. Her lifestyle is chronicled throughout the motion picture: sleeping in a coffin during the day, inability to withstand sunlight, living off the blood drained from the necks of humans with her long fangs, superior physical strength, aversion to silver, garlic and crosses, and the threat of death via stake through the heart. Recently, De Marco Studios has produced Casket Capers, a motion picture that deals with a successful married mortician who is a modern-day vampire and uses his fangs to drain the blood from the bodies entrusted to his care. The extremely strong mortician must sleep in his coffin during the day, cannot attend funerals during the day due to his inability to withstand sunlight. He has an aversion to silver, garlic and crosses. The only way the mortician can die is if someone drives a stake through his heart. Millennium Studios has filed a lawsuit against De Marco Studios for copyright infringement. Will the lawsuit succeed? (A) No, because De Marco Studios has taken only ideas from the plaintiff's motion picture. (B) Yes, because the elements at issue are nearly identical. (C) No, because both motion pictures are part of the vampire genre. (D) Yes, because the plaintiff's FangTastic! used the elements at issue before they were used by defendant's Casket Capers. Questions » Topic 1 Questions: Requirements for Copyright Protection

*(C) is the correct answer.* The elements that appear in both motion pictures are elements that are necessary for a work dealing with the genre of vampire stories. As such, they constitute scènes à faire that are not protected by copyright and may be copied without permission.

(38) Grant Jones robbed several banks in the 1990s before disappearing. He was recently captured when a tourist taking a quick, informal family picture with her cellphone in Times Square happened to accidentally capture a clear image of Jones standing on a street corner. The FBI used the tourist's picture, with her permission, to feature Jones in its Ten Most Wanted List, and this led to Jones' capture in Brooklyn within a month. Soon after, a major film and television production studio, came out with "Most Wanted: On the Trail of Grant Jones," a made-for-TV movie that chronicled the pursuit and capture of Jones. In making "Most Wanted," the studio created a highly accurate recreation of the now-famous image of Jones incidentally captured by the tourist. To re-create the tourist's photo, the studio had the actor portraying Jones in its film stand on the same street corner in New York City at the same time of day, and hired other actors who posed in the same position as the individuals featured in the tourist's photograph. The result was a photograph that looked very much like, though was identifiably different from, the tourist's photograph. The tourist sued the studio, arguing that its unauthorized recreation of the photograph infringed her copyright in the work. Who will prevail? (A) The tourist, because courts have repeatedly held that photographs are copyrightable subject matter. (B) The tourist, because the studio intentionally sought to create a photograph that was as close to the tourist's photograph as possible. (C) The studio, because it copied only unprotected facts from the photograph, not copyright-protected expression. (D) The studio, because it did not make a verbatim copy of the tourist's photograph. Questions » Topic 2 Questions: Copyrightable Subject Matter

*(C) is the correct answer.* The studio copied the physical "facts" reflected in the photograph — the fact that Jones was standing on a particular street corner in a particular place surrounded by people who looked a particular way. The studio did not copy the tourist's photograph verbatim, and the re-creation was very similar to but still clearly distinguishable from the tourist's photograph. The tourist could argue that the framing, timing, and other elements of her photograph amounted to copyrightable expression, but this is unlikely for a pair of reasons. First, this was not a carefully posed and crafted photograph such as the kind that has often been held copyrightable, e.g., Burrow-Giles Litho. Co. v. Sarony, 111 U.S. 53, 60-61 (1884), but just a "quick, informal" cell phone picture. Second, the fact that Jones was even in the photograph was an "accident," and not the result of any conscious intention by the tourist in taking the photograph.

(8) Videographer happened upon the scene of a very recent car crash involving a well-known television star, who was obviously inebriated. Videographer filmed the scene with the video function of her cell phone. The video was two minutes in length. The quality of the video's audio and images was poor. Videographer uploaded the video onto a website that permits the public to view submitted videos of celebrities in candid, often embarrassing, moments. The best argument for denying copyright protection for the video is: (A) The audiovisual work was of poor quality. (B) The length of the audiovisual work is too short for copyright protection. (C) The audiovisual work lacks sufficient originality. (D) Uploading the video onto such a public website constitutes an abandonment of copyright. Questions » Topic 1 Questions: Requirements for Copyright Protection

*(C) is the correct answer.* Videographer was not responsible for the situation she was filming, so she did not create any of the elements in front of the lens such as costuming, expression and lighting, which were the creative elements in a photograph recognized by the United States Supreme Court in Burrow-Giles Lithographic Co. v. Sarony, 111 U.S. 53 (1884). She had minimal control of the elements behind the lens such as aperture, selection of film medium, focus, audio and other automatic operations of the camera, which were the creative elements recognized in Time, Inc. v. Bernard Geis Associates, 293 F. Supp. 130 (S.D.N.Y. 1968). Videographer did select the cell phone and decided what to film and where to film it, as well as when to start and end the filming. However, a good argument could be made that these elements of originality are minimally creative in nature.

(45) Author, a noted writer of mysteries who has worked with Agent for years, trusts Agent implicitly. One day, Author sends Agent an email that reads, "I want to stop writing mysteries, and instead write an historical romance set in the Middle Ages. In this story, an earnest commoner seeks to woo a princess. The twist is that the royal family wants the princess to wed a prince, and stands in the way of their union. The princess and commoner truly love each other, so they run away, but misfortune befalls them and they die tragically. What do you think?" Agent tells Author that he should stick to mysteries, but the email inspires Agent to pen her own novel following the rough outline and plot points that Author sent her. When Agent's novel is published, Author sues Agent for copyright infringement. Will he prevail? (A) Yes, because Agent copied the plot points that Author included in the email. (B) Yes, because the material in the email was the sweat of Author's brow, and Agent's conduct amounted to wrongful misappropriation. (C) No, because Agent's copying of the material in the email was not verbatim. (D) No, because what Agent copied, the general outline of a plot in Author's email, amounted only to unprotected idea. Questions » Topic 2 Questions: Copyrightable Subject Matter

*(D) is the correct answer.* Agent did not copy the particular expression contained in Author's email, only the general plot outlines that Author had suggested. However, that plot outline was expressed at such a high level of generality that it did not amount to copyrightable subject matter. See Shipman v. RKO Pictures, 100 F.2d 533, 538 (2d Cir. 1938) (bare plots are not protected by copyright). The ideas suggested are familiar themes from world literature dating at least to Shakespeare's Romeo and Juliet: young, star-crossed lovers whose desire for each other is thwarted by venal family ambitions. Such scènes à faire (stock events and characters) are not copyrightable. See Nelson v. Grisham, 942 F. Supp. 649, 653 (D.D.C. 1996), aff'd, 132 F.3d 1481 (D.C. Cir. 1997). (A) is incorrect because it misstates the facts of the question. Agent did not copy the particular expression contained in the email. Rather, Agent merely relied on the general ideas contained in the email when creating her own subsequent work. (B) is incorrect because copyright law does not protect a work simply because it is created from the "sweat of the brow." This theory was rejected by the Supreme Court in Feist Publications, Inc. v. Rural Telephone Service Company, Inc., 499 U.S. 340, 359 (1991). (C) is incorrect because although verbatim copying is evidence of infringement, it is not required to show infringement. The traditional test of the infringement of the reproduction right is whether the defendant's work is substantially similar, not verbatim.

(51) Archaeologist publishes a blockbuster article in which he reveals that he has discovered a new store of Mayan artifacts deep in the Yucatan, and argues persuasively that this discovery supports the theory that Mayan civilization began several centuries earlier than previously believed. The article draws a host of strong reactions. For example, Amateur runs a website devoted to archaeology as a hobby, in which he publishes a brief news item entitled Big Discovery in the Yucatan. Amateur's feature recounts the details of Archaeologist's discovery, listing where it happened, what techniques were used in the excavation, and what was uncovered. Subsequently, Archaeologist's longtime academic rival, Nemesis, publishes an article in which he describes Archaeologist's theory about the dawn of Mayan civilization, using Nemesis' own words, and then explains why he thinks the theory is bunk. Archaeologist is incensed at both Amateur and Nemesis, and sues each of them for infringing the copyright in his original article. Will Archaeologist prevail? (A) Yes, Archaeologist will prevail, but only against Amateur, because Nemesis did not copy any protectable subject matter. (B) Yes, Archaeologist will prevail, but only against Nemesis, because Amateur did not copy any protectable subject matter. (C) Yes, Archaeologist will prevail against both Amateur and Nemesis, because both of them copied protectable subject matter. (D) No, Archaeologist will not prevail against either Amateur or Nemesis, because neither of them copied protectable subject matter. Questions » Topic 2 Questions: Copyrightable Subject Matter

*(D) is the correct answer.* Neither Amateur nor Nemesis copied copyrightable subject matter. Where Archaeologist's dig took place and what was discovered are scientific and historical facts, which are not copyrightable, so there is no infringement claim against Amateur's reporting these facts on his website. See Rosemont Enters., Inc. v. Random House, Inc., 366 F.2d 303 (2d Cir. 1966). Similarly, the interpretation of those historical and scientific facts is also not copyrightable, Hoehling v. Universal City Studios, Inc., 618 F.2d 972 (2d Cir. 1980), so the element of Nemesis' article that recounts Archaeologist's interpretation of his discoveries also does not amount to an infringement. Had Nemesis copied the particular words Archaeologist used when articulating his new theory, that would likely amount to infringement, but the facts specify that Nemesis did not do this, but instead expressed Archaeologist's theory entirely in his own words. (A), (B), and (C) are incorrect because they wrongly assume that Nemesis, Amateur, or both, took copyrighted material.

(34) Style Over Substance is a black-and-white motion picture, produced by Studio, that is no longer protected by copyright. Network has invested the necessary funds to colorize the motion picture without Studio's permission. Each pixel of each frame of the motion picture was colored from a palette of 16 million colors. Network has issued a press release stating that any broadcast or cable television channel or station that transmits Style Over Substance without Network's permission will be liable to Network for copyright infringement. Is Network's claim accurate? (A) Yes, because the colorized version of Style Over Substance contains all of the elements of the original motion picture. (B) Yes, because Network has a copyright in the colorized version of Style Over Substance. (C) No, because Style Over Substance is in the public domain. (D) No, because the original version of Style Over Substance may be used without Network's permission. Questions » Topic 1 Questions: Requirements for Copyright Protection

*(D) is the correct answer.* Network's claim of copyright protection of the colorized version of Style Over Substance is limited to the original new elements that make it a protected derivative work. The portions of the black and white version of the motion picture that entered the public domain remain in the public domain.

(47) Student is a sophomore at the University of Spoiled Children, where he is a member of the biggest fraternity on campus. One night, as a gag, Student takes the University's campus directory (which includes names, phone numbers, and campus addresses of all 2,500 of the students currently enrolled at the University, arranged by year and address), and publishes "Student's Guide to the Most Popular People on Campus" in a printed leaflet. The guide takes 50 of the listings from the campus directory verbatim, reflecting the individuals who Student deems to be the most popular on campus, and lists them in alphabetical order. The University claims to own the copyright in the campus directory, and, not amused by the gag, sues Student for infringing its copyright in the directory. Will the University succeed? (A) Yes, the University will succeed because Student copied its protectable expression verbatim. (B) Yes, the University will succeed because its copyright extends to all student directories for its enrollees, regardless of how they are arranged. (C) No, the University will not succeed because directories are in no way copyrightable. (D) No, the University will not succeed because Student copied only unprotectable elements of the directory (information) and nothing that could be copyrighted (such as selection or arrangement). Questions » Topic 2 Questions: Copyrightable Subject Matter

*(D) is the correct answer.* Student took only generally available facts, such as names, addresses, and phone numbers, from the University's directory. Facts are not copyrightable. See Harper & Row Publishers, Inc. v. Nation Enters., 471 U.S. 539, 556 (1985) ("No author may copyright . . . the facts he narrates."). While the University may have had a claim if Student had copied any original selection or arrangement of these facts, that was not the case here, because Student used only the facts in the University's directory and infused them with his own selection and arrangement. (A) is incorrect because while Student copied certain elements of the directory verbatim, those elements were uncopyrightable facts, not protectable expression. (B) is incorrect because the University's copyright in the directory is actually very thin. See Kregos v. Associated Press, 937 F.2d 700 (2d Cir. 1991). It extends no farther than the selection and arrangement of facts, and certainly does not include "the idea of all student directories for their employees." (C) is incorrect because some elements of directories, such as their selection and arrangement of facts, are copyrightable. 17 U.S.C. § 103(a) (compilations are copyrightable); see, e.g., Feist Publications, Inc. v. Rural Tel. Serv. Co., Inc., 499 U.S. 340, 357 (1991). Answers » Topic 2 Answers: Copyrightable Subject Matter

(5) One recent evening in Los Angeles, Comedian included in her act at a comedy club a 10-minute extemporaneous original comedy routine concerning the recent firing of a government official. The content of the routine was created on the spur of the moment. Comedian had not written out the material, nor did she record the routine when she presented the material at the comedy club. Is Comedian's extemporaneous comedy routine protected? (A) The routine is protected by federal copyright because a 10-minute comedy routine undoubtedly includes expression as well as ideas. (B) The routine is protected by federal copyright because it is a literary work. (C) The routine is protected by state common-law copyright because it was merely a performance. (D) The routine is protected by state common-law copyright because a 10-minute comedy routine undoubtedly includes original expression as well as ideas. Questions » Topic 1 Questions: Requirements for Copyright ProtectionOne

*(D) is the correct answer.* The comedy routine is protected by California's state common-law copyright because it is original expression that has not been fixed. A comedy routine that is 10 minutes in length will undoubtedly include protectable expression. Cal. Civ. Code § 980(a)(1).

(12) A work has been fixed in a physical disc. Has it been fixed in a copy or a phonorecord? (A) Copy. (B) Phonorecord. (C) Neither a copy nor a phonorecord. (D) It depends. Questions » Topic 1 Questions: Requirements for Copyright Protection

*(D) is the correct answer.* The determination of whether the object in which a work is fixed is a copy or a phonorecord is dependent on what type of work is fixed. Phonorecords are material objects in which sounds, other than those accompanying an audiovisual work, are fixed and from which the sounds can be perceived, reproduced, or otherwise communicated, either directly or with the aid of a machine or device. 17 U.S.C. § 101; see ABKCO Music, Inc. v. Stellar Records, Inc., 96 F.3d 60 (2d Cir. 1996) (karaoke CD-ROM on which sounds and visual representations of lyrics were fixed was a copy, not a phonorecord). Copies are material objects, other than phonorecords, in which a work is fixed and from which the work can be perceived, reproduced, or otherwise communicated, either directly or with the aid of a machine

(40) Historian has written a short history of the City of San Angeles. He obtained the facts contained in the short history from public records and newspaper articles published by the San Angeles Bee, as well as from Historian's personal knowledge. Historian has not received the permission of the San Angeles Bee to use any of its articles. The short history of the City of San Angeles is: (A) Not copyrightable because it is a factual work. (B) Not copyrightable because it is an unauthorized derivative work. (C) Copyrightable, but only if the short history was written prior to 1978. (D) Copyrightable, though the copyright protection it receives may be more limited than the copyright protection given to a work of fiction. Questions » Topic 2 Questions: Copyrightable Subject Matter

*(D) is the correct answer.* The expression of factual information is protected by copyright. Factual works, however, are more susceptible to a claim of fair use than are works of fiction. Furthermore, there may be more limitations on the protection given to the organization and structure of a nonfiction literary work. (A) is incorrect. Although facts are not protected by copyright, the original expression of facts is protected. For example, the Encyclopedia Britannica is protected by copyright because its entries contain original expression as well as factual information. (B) is incorrect. The short history would not be deemed a derivative work of the newspaper articles because only unprotected facts were taken from the articles, therefore, the short history cannot be said to constitute a derivative work as it did not substantially copy protectable expression from the prior work. (C) is incorrect. The nonprotection of facts and the protection of the expression of facts by copyright have remained constant before, during and after 1978. Answers » Topic 2 Answers: Copyrightable Subject Matter

(61) Consider the facts of question 60. User buys a new laptop computer and is disappointed to discover that the version of DethKlok 2000 he purchased is no longer compatible with the operating system on his new computer. User searches the Internet and finds that by making a few modifications to the source code of the DethKlok 2000 program he can render the application compatible with his new laptop. User modifies the source code and the alteration successfully renders DethKlok 2000 compatible with his new computer. Does User's alteration of the DethKlok 2000 program amount to copyright infringement? (A) Yes. User's alteration of the source code that comprises DethKlok 2000 amounts to an unauthorized derivative work, violating the copyright owner's exclusive right of adaptation. (B) Yes. User's creation of what is essentially a new version of DethKlok 2000 violates the copyright owner's exclusive right of reproduction. (C) No. User's ownership of the DethKlok 2000 program entitles him to alter the source code that comprises that program in any way he sees fit. (D) No. While User's alteration of the source code that comprises the DethKlok 2000 program is an unauthorized adaptation of a copyrighted work of authorship, owners of computer programs are entitled to make such alterations in order to make applications compatible with new hardware. Questions » Topic 2 Questions: Copyrightable Subject Matter

*(D) is the correct answer.* The owner of a computer program is permitted to make an adaptation of the computer program, without authorization from the copyright owner, as an essential step in the utilization of the computer program. 17 U.S.C. § 117; see Aymes v. Bonelli, 47 F.3d 23, 26-27 (2d Cir. 1995) (adaptation permitted where the changes were necessary to keep program current and to maintain its viability with an upgraded computer system). (A) is incorrect. While User's alteration of the source code that comprises DethKlok 2000 does create an unauthorized derivative work, this particular conduct falls within a narrow statutory exemption from liability. (B) is incorrect. Even if User made a copy of the DethKlok 2000 source code in revising it, that reproduction would also fall within the section 117 statutory exemption. (C) is incorrect. User's ownership of an authorized copy of DethKlok 2000 does not obviate the adaptation right enjoyed by the owner of the copyright in the source code that comprises the program. It merely permits a limited exception to the copyright owner's adaptation right. Answers » Topic 2 Answers: Copyrightable Subject Matter

(41) Musical Group recently performed six musical works in a nightclub, pursuant to a public performance license obtained by Club Owner. Club Owner filmed the performance with a digital camera with the permission of Musical Group, which also agreed to its release on DVD. Musical Group has entered into a contract with Record Company, under which Record Company is assigned the copyright in all sound recordings created by Musical Group. After the release of Musical Group's performance on DVD, Record Company seeks compensation for the unauthorized distribution of its sound recordings on the DVD. Record Company's claim will be: (A) Successful, because Record Company owns the copyrights in the sound recordings contained in the DVD release. (B) Successful, because Club Owner's public performance license only addresses the performance of musical compositions, not the creation of sound recordings. (C) Unsuccessful, because there is no privity of contract between Club Owner and Record Company. (D) Unsuccessful, because the DVD release contains an audiovisual work, not a sound recording. Questions » Topic 2 Questions: Copyrightable Subject Matter

*(D) is the correct answer.* The sound contained in the DVD accompanies a series of related images. As such, the work fixed in the DVD is an audiovisual work, not a sound recording. (A) is incorrect. As explained in the answer above, the DVD contains an audiovisual work, not sound recordings. (B) is incorrect. The statement of the law is correct, but not relevant to Record Company's incorrect claim that the filmed performance constituted a sound recording. (C) is incorrect. If the DVD did contain sound recordings owned by Record Company, the absence of any contractual relationship between Club Owner and Record Company would prohibit Club Owner from distributing the copyright protected sound recordings without the permission of Record Company. However, the DVD contained only six musical works and an audiovisual work, not sound recordings.

(62) Printer owns a printing shop. One day, he invents a new typeface script called Lucinda that allows for more characters to appear on a page without sacrificing readability. Printer does a brisk business formatting documents with his new typeface, and refuses to license Lucinda to other printers or to makers of word processing software so he can capture all the value associated with the font. Printer discovers that an exact copy of Lucinda has been added to the latest edition of a word processing program produced and marketed by the software company MacroSoft. Printer wants to sue MacroSoft for its unauthorized appropriation of his lucrative original font. Will he succeed? (A) Yes. Fonts are protectable as pictorial or graphical works. (B) Yes. Fonts are protectable as literary works. (C) No. Fonts cannot possess the requisite degree of originality to merit copyright protection. (D) No. Regardless of their originality, fonts or typefaces cannot be protected by federal copyright. Questions » Topic 2 Questions: Copyrightable Subject Matter

*(D) is the correct answer.* Though some typefaces may have sufficient originality to be copyrightable, Congress, as a matter of public policy, has determined that the design of typefaces will not be protected by copyright, though they may be protected by the law of unfair competition. See Monotype Corp. PLC v. Int'l Typeface Corp., 43 F.3d 443 (9th Cir. 1994); Eltra Corp. v. Ringer, 579 F.2d 294 (4th Cir. 1978). (A) and (B) are incorrect; whether pictorial, graphical, or literary works, fonts are not protectable by federal copyright. (C) is incorrect. Fonts could certainly possess the requisite degree of originality to merit copyright protection. Although the letters of the alphabet themselves are in the public domain, they could be depicted with enough creative expression that the result easily crosses the low bar of originality. Answers » Topic 2 Answers: Copyrightable Subject Matter

(26) Hunter creates realistic turkey decoys for a living. He has recently noticed that the turkey decoys sold by Competitor are substantially similar, although not exactly identical, to his decoys. Will Hunter be successful if he brings a copyright infringement action against Competitor and can show that Competitor had access to Hunter's decoys? (A) Yes, because sculptural works are copyrightable subject matter. (B) Yes, because the reproduction right protects against copies that are substantially similar. (C) No, because anyone can copy items found in nature. (D) No, because depictions of natural objects have limited copyright protection. Questions » Topic 1 Questions: Requirements for Copyright Protection

*(D) is the correct answer.* Where the subject matter of the copyright is a depiction of an object that appears in nature, the scope of copyright protection is limited. To prove copyright infringement, the owner of the copyright must prove that the defendant's work is a verbatim or near-verbatim copy of the plaintiff's work. In this case, Hunter is only able to show that Competitor's decoys are substantially similar. See Streeter v. Rolfe, 491 F. Supp. 416 (W.D. La. 1980); Satava v. Lowry, 323 F.3d 805, 812-13 (9th Cir. 2003) (realistic depictions of live animals were "first expressed by nature, are the common heritage of humankind, and no artist may use copyright law to prevent others from depicting them"); Franklin Mint Corp. v. National Wildlife Art Exchange, Inc., 575 F.2d 62 (3d Cir. 1978) (paintings of cardinals). But see Mattel, Inc. v. Goldberger Doll Mfg. Co., 365 F.3d 133, 135-36 (2d Cir. 2004) ("There are innumerable ways of making upturned noses, bow lips, and widely spaced eyes [for dolls]."); Hamil Am., Inc. v. GFI, 193 F.3d 92, 100-01 (2d Cir. 1999) (where the plaintiff's work is a stylized floral pattern, mere substantial similarity need be shown).


Kaugnay na mga set ng pag-aaral

Animal Science 101L Exam Questions

View Set

Chapter 2: Something You Are - Biometric Methods

View Set

Chapter 4: Network Access - Intro. to Networks v6

View Set

Natural Disaster in a Small Community - HESI RN Case Studies

View Set